You are on page 1of 63

Basic Science 2018

Question 1 of 100
What is the most likely mechanism that causes the thiazolidinedione group of
antihyperglycemic medications to increase the risk of fractures? These medications

1. decrease the activity of receptor activator of nuclear factor kappa-B ligand


(RANKL).
2. increase tumor necrosis factor alpha (TNF-α).
3. suppress osteoblastic transcription factors.
4. decrease osteoprotegerin.
Discussion : 3

Thiazolidinedione medications activate peroxisome proliferator-activated receptor ƴ


(PPARy). PPARƴ activation leads to the differentiation of mesenchymal stem cells
towards the adipocyte lineage. Thiazolidinediones are reported to decrease the
osteblastogenic transcription factors such asrunt-related transcription factor 2.
Suppression of osteoblastic transcription factors is the most commonly accepted
mechanism for the increased rate of fractures in patients taking thiazolidinediones.

Question 2 of 100
Figures 1 through 3 display radiographs and an MRI from a 29-year-old man who has
complained of left leg pain since sustaining a gunshot wound 18 months ago. He denies
any fevers or chills but does have pain and drainage from his wound. His erythrocyte
sedimentation rate (ESR) is 105 mm/h (reference range: 0-20 mm/h), C-reactive protein
(CRP) level is 12 mg/L (reference range: 0.08-3.1 mg/L), white blood cell (WBC) count
is 8,000 /µL (reference range: 4500-11000 /µL), and vitamin D level is 15 ng/mL
(reference range: 20-40ng/ml).

A major reason for implant removal is the formation of a bacterial biofilm that is
extremely difficult to eradicate. This biofilm mostly likely includes bacteria in what
stage?
1. Planktonic
2. Sessile
3. Rapid-proliferation
4. Metabolic

Discussion: 2
This patient has chronic osteomyelitis and infected nonunion of the tibia. The bone
indicated by the arrow is sequestrum, or infected, dead bone. Involucrum is new bone
laid down around the infection. Treatment of this infection requires debridement and
local antibiotic delivery, often accomplished with an antibiotic-impregnated cement. The
elution characteristics often vary by cement type, but generally, peak concentrations are
reached in 24 hours, and the concentrations can remain bactericidal for 4 months.

Question 3 of 100
CLINICAL SITUATION

Figures 1 through 3 display radiographs and an MRI from a 29-year-old man who has
complained of left leg pain since sustaining a gunshot wound 18 months ago. He denies
any fevers or chills but does have pain and drainage from his wound. His erythrocyte
sedimentation rate (ESR) is 105 mm/h (reference range: 0-20 mm/h), C-reactive protein
(CRP) level is 12 mg/L (reference range: 0.08-3.1 mg/L), white blood cell (WBC) count
is 8,000 /µL (reference range: 4500-11000 /µL), and vitamin D level is 15 ng/mL
(reference range: 20-40ng/ml).

The mechanism of action for vancomycin and tobramycin differ, in that vancomycin is
1. concentration-dependent, killing by inhibiting cell wall synthesis.

2. concentration-dependent, killing by binding the 30S ribosomal subunit.

3. time-dependent, killing by inhibiting cell wall synthesis.


4. time-dependent, killing by binding the 30S ribosomal subunit.

Discussion : 3

Vancomycin is a common antibiotic used in cement delivery. Its mechanism of action


disrupts cell-wall synthesis. It also kills bacteria in a time-dependent manner, not in a
concentration-dependent way like tobramycin and other aminoglycosides. This means
that, for the drug to be effective, the concentration must be above the minimal inhibitory
concentration between doses, illustrating why following a vancomycin trough is
important. Drugs that kill in a concentration-dependent way still have effect, even after a
limited exposure; this fact explains why the dosing for gentamycin is recommended once
per day for open fractures

Question 4 of 100
A 2-year-old girl is seen in the emergency department. Her main symptom is swelling of
the right knee and refusal to bear weight. On examination, the child is febrile to 39.0°C,
her right knee is mildly swollen, and she resists attempts at passive motion. The patient’s
white blood cell count is 10,500 cells per milliliter, her C-reactive protein level is normal,
and her erythrocyte sedimentation rate is mildly elevated at 45 mm per hour. An MRI
reveals a moderate effusion but no evidence of osteomyelitis. A knee aspirate reveals a
white blood cell count of 60,000 cells per milliliter. To increase the likelihood of
isolating the causative bacteria, what culture medium should be utilized?
1. Eosin methylene blue agar
2. Granada agar
3. Blood agar
4. Mannitol salt agar

Discussion : 3
Kingella kingae is increasingly recognized as an important cause of osteoarticular
infections, especially in patients between the ages of 6 months and 4 years. K kingae is a
fastidious gram-negative coccobacillus and is part of the normal oral pharangeal flora in
young children. Infections due to K kingaefrequently present in a delayed fashion with
subacute symptoms. In one study, the mean length of symptoms prior to presentation was
9 days. Additionally, only 15% of patients were febrile on admission. The true incidence
of infections associated with K kingae may be considerably higher than previously
thought, given the difficulty of growing K kingae on standard culture media. In one
study, only 10% of confirmed cases of osteoarticular infections associated with K
kingae displayed growth on standard solid culture media. Blood agar is recommended for
patients suspected of K kingaeinfection. Additionally, polymerase chain reaction has
shown promise as an accurate and quick diagnostic tool; however, it is not widely
available. Eosin methylene blue agar is toxic to gram-positive bacteria but is used to
isolate coliforms. Granada agar is used to isolate group B Streptococcus species.
Mannitol salt agar is specific for gram-positive bacteria, especially mannitol fermenters.

Question 5 of 100
A 45-year-old man sustains a closed midshaft transverse tibial fracture following a skiing
injury. What is the biggest risk factor for nonunion following surgical treatment of the
injury?

1. NSAID use
2. Smoking
3. Body mass index (BMI)
4. Age

Discussion : 2

In several studies and a large meta-analysis, smoking has been proven to increase the
odds of nonunion, having an increased odds ratio of 2.16, and overall delay in healing in
tibial fractures. NSAID use also can lead to nonunion or delayed union, but smoking is
more likely to do so. Age and BMI can affect healing, but smoking consistently
demonstrates a negative effect.
Question 6 of 100
Recombinant bone morphogenetic protein-2 (BMP-2) plays a role in what stage of
fracture healing?

1. Traumatic
2. Inflammatory
3. Reparative
4. Remodeling

Discussion: 2

Following traumatic injury to bone, BMPs and other proinflammatory cytokines signal
the initiation of fracture healing. BMP-2 does so by recruiting mesenchymal stem cells to
the site of injury.

Question 7 of 100
CLINICAL SITUATION

Figures 1 and 2 show radiographs from a 12-year-old girl with a several-year history of
multiple bony protuberances. On examination, she has palpable masses about the bilateral
knees, wrists, and shoulders.

Histologic analysis of this patient’s physis would reveal a reduction of what


proteoglycan?

1. Chondroitin sulfate
2. Dermatan sulfate
3. Keratan sulfate
4. Heparan sulfate

Discussion: 4

The radiographs demonstrate multiple osteochondromas or exostoses about the distal


femur and proximal tibia. The patient’s most likely diagnosis is hereditary multiple
exostoses (HME). The average age at diagnosis is 3 years old, and 94% of patients have
exostoses about the knee. Other common sites include the proximal humerus, scapula,
wrist, and proximal femur. The vast majority of patients display an autosomal dominant
inheritance pattern, although spontaneous mutations have been described.

In most patients, the underlying genetic abnormality involves the EXT1 or EXT2 gene,
located on chromosomes 8 and 11, respectively. These genes encode glycosyltransferases
involved in the synthesis of heparan sulfate. Heparan sulfate plays an important role in
physeal signaling, and chondrocyte disorganization leads to exostosis development.
Chondroitin sulfate, dermatan sulfate, and keratan sulfate are other proteoglycans, but
they are not involved in the pathogenesis of HME.

The lifetime risk of malignant transformation to chondrosarcoma in patients with HME


has been reported to range between 1% and 25%; however, more recent studies have
placed the risk between between 3% and 7%. The most recent study demonstrated a 3%
risk of malignant transformation at an average age of 29 years. Malignant transformation
can occur in an exostosis in any location; however, pelvic exostoses have been shown to
have a higher rate of malignant transformation.

The risk of malignant transformation has been noted to be higher in disorders involving
multiple enchondromas, including Mafucci syndrome and Ollier disease. Some studies
have demonstrated a nearly 100% risk of malignant transformation in Mafucci syndrome.

Question 8 of 100
A 45-year-old woman is being seen for an evaluation of metabolic bone disease because
of a history of multiple fractures in the past 2 years. Laboratory data show calcium: 9
mg/dL (reference range: 8.2-10.2 mg/dL), phosphorus: 1.5 mg/dL (reference range: 2.3-
4.7 mg/dL), parathyroid hormone: 30 pg/mL (reference range: 10-65 pg/mL), l,25
hydroxy vitamin D: 18 ng/ml (reference range: 25-45 ng/mL). She recently was
diagnosed with osteoblastoma in the thoracic spine. What factor is responsible for the
laboratory abnormalities seen in this patient?

1. FGFR3 gene mutation


2. Fibroblast growth factor 23 (FGF23) excess
3. Transforming growth factor beta (TGF-β) excess
4. Activation of receptor activator of nuclear factor kappa-B ligand (RANKL)
Discussion : 2

This patient has oncogenic osteomalacia. Laboratory data show hypophosphatemia and
low 1,25 hydroxy vitamin D levels. Oncogenic osteomalacia is seen in association with
mesenchymal tumors, especially osteoblastoma, fibrous dysplasia, and fibromas.
Osteomalacia is caused by the overactivity of FGF23, which decreases phosphate
reabsorption in the kidneys, causing hypophosphatemia. It also inhibits the activity of 1
alpha hydroxylase (responsible for converting 25 hydroxy vitamin D to 1,25 hydroxy
vitamin D), causing decreased levels of 1,25 hydroxy vitamin D. FGFR 3 activating
mutation causes achondroplasia. TGF-β excess is seen in patients with Marfan syndrome.

Question 9 of 100
Nonsurgical treatment has failed in a 64-year-old woman with a history of low back pain
and bilateral lower extremity pain. She has been indicated for posterolateral lumbar
decompression and fusion. Imaging studies reveal dynamic instability of an L4 -5 grade 1
degenerative spondylolisthesis with severe spinal stenosis. During routine medical
clearance evaluation, the patient’s history reveals a screening bone mineral density of
65% of the young adult mean. Administration of which adjunct treatment has been shown
to enhance the rate of fusion in such patients?

1. Calcitonin
2. Teriparatide
3. Denosumab
4. Alendronate
Discussion: 2

Osteoporotic patients undergoing spinal fusion procedures are at greater risk for
procedurally associated complications, including pseudarthrosis, instrumentation failure,
or even fracture. Recombinant parathyroid hormone has anabolic properties for the
stimulation of bone growth, which has been shown to be beneficial in postmenopausal
women undergoing spinal fusion. Although the other agents are used for the treatment of
osteoporosis, perioperative administration has not been shown to enhance spinal fusion
rates.

Question 10 of 100
Figures 1 through 3 show radiographs from a 32-year-old man who sustained a closed
both-bone forearm fracture. Figures 2 and 3 were taken 6 weeks postoperatively. A
histological sample of the fracture site at this point would most likely show

1. mesenchymal cells laying fibrous tissue and inflammatory cells.


2. chondrocytes and cartilage undergoing endochondral ossification.
3. osteoclasts and osteoblasts remodeling lamellar bone.
4. osteoblastic formation of woven bone.

Discussion: 3

The radiographs reveal a fracture that is going through primary fracture healing with rigid
stability and compression. No callus formation is present. All the answer choices describe
events occurring in secondary fracture healing, except the osteoclast and osteoblast
remodeling. In this process, cutting cones are formed. Osteoclasts form the head of the
cone, and osteoblasts lay down new bone behind.
Question 11 of 100
A patient has a dorsal hand abscess. Irrigation and debridement are performed, and
cultures are obtained. Then, 48 hours later, the cultures grow methicillin-
resistant Staphylococcus aureus (MRSA) that are resistant to erythromycin. Because of
inducible resistance, a D-Zone test should be requested before ordering which antibiotic?

1. Vancomycin
2. Clindamycin
3. Trimethoprim-sulfamethoxazole
4. Daptomycin

Discussion: 2
Antibiotic resistance is an important issue with MRSA. Although all the above choices
treat MRSA, each choice has advantages and disadvantages. Isolates of MRSA that are
resistant to erythromycin have been shown to become resistant to clindamycin, a process
called inducible resistance, which is conferred by a plasmid that alters the 50S ribosome
binding site for both clindamycin and erythromycin

Question 12 of 100
Anti-sclerostin antibody increases bone formation by targeting what molecular pathway?

1. Wnt
2. Bone morphogenetic protein (BMP)
3. Notch
4. Indian hedgehog

Discussion: 1

All of the listed factors serve critical functions in bone formation. Only BMP -based
therapies currently are FDA approved, however. Sclerostin is an extracellular antagonist
of Wnt signaling and, to a lesser extent, BMP signaling. Blosozumab currently is an
investigational antibody therapy designed to block sclerostin’s ability to inhibit Wnt
signaling, netting a positive effect on bone formation. In a phase 2 trial involving 120
postmenopausal women, 1 year of blosozumab treatment resulted in a 17% increase in
bone mineral density in the lumbar spine.
Question 13 of 100
Figures 1 through 4 show the AP radiograph, axial CT, coronal T2-weighted MRI, and
biopsy specimen from a 73-year-old man with severe left hip pain. In this patient’s
disease, osteoblastic bone formation is suppressed by

1. tumor necrosis factor


2. receptor activator of nuclear factor kappa-B ligand (RANKL)
3. interleukin-6.
4. Bence-Jones proteins.

Discussion: 1

The AP radiograph and axial CT of the pelvis reveal a lucent lesion in the supra-
acetabular region. The T2-weighted coronal MRI reveals the intramedullary extent of the
disease. The biopsy specimen shows scattered plasma cells consistent with the diagnosis
of multiple myeloma. Major mediators of osteoclastogenesis in multiple myeloma
include RANKL, interleukin-6, and macrophage inflammatory protein-1 alpha.
Osteoblastic bone formation is suppressed by tumor necrosis factor and Dickkopf-1
(DKK-1). Investigations are underway to find agents that improve osteoblast
differentiation by modulating inhibitors such as DKK-1 to enhance the structural integrity
of involved bones.

Question 14 of 100
CLINICAL SITUATION

Figures 1 and 2 show radiographs from a 12-year-old girl with a several-year history of
multiple bony protuberances. On examination, she has palpable masses about the bilateral
knees, wrists, and shoulders.
What is the inheritance pattern of the patient’s most likely diagnosis?

1. Autosomal dominant
2. Autosomal recessive
3. X-linked recessive
4. X-linked dominant

Discussion: 1

The radiographs demonstrate multiple osteochondromas or exostoses about the distal


femur and proximal tibia. The patient’s most likely diagnosis is hereditary multiple
exostoses (HME). The average age at diagnosis is 3 years old, and 94% of patients have
exostoses about the knee. Other common sites include the proximal humerus, scapula,
wrist, and proximal femur. The vast majority of patients display an autosomal dominant
inheritance pattern, although spontaneous mutations have been described.

In most patients, the underlying genetic abnormality involves the EXT1 or EXT2 gene,
located on chromosomes 8 and 11, respectively. These genes encode glycosyltransferases
involved in the synthesis of heparan sulfate. Heparan sulfate plays an important role in
physeal signaling, and chondrocyte disorganization leads to exostosis development.
Chondroitin sulfate, dermatan sulfate, and keratan sulfate are other proteoglycans, but
they are not involved in the pathogenesis of HME.

The lifetime risk of malignant transformation to chondrosarcoma in patients with HME


has been reported to range between 1% and 25%; however, more recent studies have
placed the risk between between 3% and 7%. The most recent study demonstrated a 3%
risk of malignant transformation at an average age of 29 years. Malignant transformation
can occur in an exostosis in any location; however, pelvic exostoses have been shown to
have a higher rate of malignant transformation.
15 of 100
A 57-year-old man with chronic kidney disease who recently began undergoing dialysis
treatment five times per week comes to your office for a 3-month follow-up appointment
after a right total knee replacement. The patient is at risk of renal osteodystrophy because
of the failure of vitamin D conversion to which of the following active metabolites?

1. Ergocalciferol (vitamin D2)


2. Cholecalciferol (vitamin D3)
3. 24, 25-dihydroxycholecalciferol
4. Calcitriol (1,25-dihydroxycholecalciferol)

Discussion: 4

Calcitriol is the active metabolite of vitamin D that is formed by 1 alpha-hydroxylase


from calcidiol. Secondary hyperparathyroidism is a condition commonly seen in patients
with chronic kidney disease characterized by hyerphosphatemia, hypocalcemia, high
parathyroid hormone levels, and a reduced amount of calcitriol from damage to the
kidney.

Question 16 of 100
Figures 1 and 2 show the radiographs from a 10-year-old female gymnast with a 3-month
history of wrist swelling and pain. During competition, her pain worsens acutely, and she
now reports reduced range of motion. What is the location of the injury?

1. Zone of provisional calcification


2. Hypertrophic zone
3. Proliferative zone
4. Germinal layer

Discussion: 1

Skeletally immature athletes are at risk for injuries not seen in adults because of
differences in the structure of the growing bone and physis. Growth plates are susceptible
to shear injury at the epiphyseal-metaphyseal junction. Epiphyseal injuries present with
persistent or severe pain, deformity, or the inability to move a joint. Typically, physeal
injuries occur from repetitive loading that disrupts the metaphyseal perfusion, inhibiting
ossification in the zone of provisional calcification. The hypertrophic zone widens as
chondrocytes continue to transition from the germinal layer to the proliferative zone.
Widening of the physis may be seen radiographically

Question 17 of 100
Current strategies for treating the aberrant bone remodeling that occurs in osteoporosis
involve using agents that prevent bone resorption or stimulate bone formation. Agents
that stimulate bone formation include

1. bisphosphonates.
2. denosumab.
3. calcitonin.
4. parathyroid hormone

Discussion : 4

Parathyroid hormone binds to receptors on osteoblasts and preosteoblasts and thereby


stimulates bone formation when dosed intermittently. Bisphosphonates, denosumab, and
calcitonin are all bone antiresorptive agents. They primarily have inhibitory effects on
osteoclasts, each by different molecular mechanisms

Question 18 of 100
What is the most abundant matrix protein produced by osteoblasts?

1. Type I collagen
2. Receptor activator of nuclear factor kappa-B (RANK)
3. Aggrecan
4. Interleukin-1 (IL-1)

Discussion: 1

Type I collagen is the major matrix protein produced by osteoblasts and is an essential
component of bone. RANK is a cell receptor that is important in the regulation of
osteoclasts. Aggrecan is an important proteoglycan component of cartilage. IL-1 is an
inflammatory factor.

Question 19 of 100
In the context of joint arthroplasty, the alpha-defensin immunoassay test is useful for the
detection of

1. aseptic loosening.

2. metal corrosion.

3. periprosthetic infection.
4. bone ingrowth.

Discussion : 3

Alpha-defensin is a protein released by activated neutrophils in response to infection. The


detection of alpha-defensin in synovial fluid is highly sensitive and specific as a marker
of periprosthetic infection. Aseptic loosening and the bone ingrowth of prostheses
generally are detected radiographically. Metal corrosion issues—in metal-on-metal
prostheses, for example—generally are followed using blood metal ion levels.

Question 20 of 100
A 10-year-old child arrives at the emergency department with pain in the leg and a limp.
An MRI shows acute osteomyelitis of the tibia without abscess formation. Blood cultures
are positive for Staphylococcus aureus. The patient is admitted and treated with
intravenous antibiotic therapy. The next day, shortness of breath develops, and CT shows
multiple pulmonary emboli. What gene in S aureus is responsible for the clinical
deterioration of this patient?

1. Gene involved in biofilm formation


2. Penicillinase gene
3. Panton-Valentine Leukocidin (PVL) gene
4. Alteration in penicillin binding protein gene

Discussion :3

The clinical deterioration seen in this patient mainly results from septic pulmo nary
emboli caused by septic deep vein thrombophlebitis. Septic deep vein thrombophlebitis
can be seen as a complication of osteomyelitis in children. It more often is seen with
tibial osteomyelitis. Leukocidin, a product of the PVL gene in the bacteria, has been
reported to be responsible for this complication. Biofilm formation causes adherence to
implants. Penicillinase and penicillin binding protein genes are responsible for drug
resistance.
Question 21 of 100
CLINICAL SITUATION

Figures 1 through 3 display radiographs and an MRI from a 29-year-old man who has
complained of left leg pain since sustaining a gunshot wound 18 months ago. He denies
any fevers or chills but does have pain and drainage from his wound. His erythrocyte
sedimentation rate (ESR) is 105 mm/h (reference range: 0-20 mm/h), C-reactive protein
(CRP) level is 12 mg/L (reference range: 0.08-3.1 mg/L), white blood cell (WBC) count
is 8,000 /µL (reference range: 4500-11000 /µL), and vitamin D level is 15 ng/mL
(reference range: 20-40ng/ml).

After thorough debridement, the surgeon wishes to place an antibiotic-coated


intramedullary nail made of polymethylmethacrylate (PMMA), plus vancomycin and
tobramycin coat the nail by mixing them in the PMMA cement. Although in vivo
concentrations of the antibiotics often peak during the first 24 hours, up to how many
months can the concentrations be expected to remain bactericidal?

1. 1 3. 8
2. 4 4. 12
Discussion: 2

This patient has chronic osteomyelitis and infected nonunion of the tibia. The bone
indicated by the arrow is sequestrum, or infected, dead bone. Involucrum is new bone
laid down around the infection. Treatment of this infection requires debridement and
local antibiotic delivery, often accomplished with an antibiotic-impregnated cement.
The elution characteristics often vary by cement type, but generally, peak
concentrations are reached in 24 hours, and the concentrations can remain bactericidal
for 4 months.

Question 22 of 100
In what zone of the articular cartilage are the chondrocytes arranged in columns and the
collagen fibers oriented vertical to the articular surface?

1. Calcified zone

2. Middle zone

3. Superficial zone

4. Deep zone

Discussion: 4

Four zones are identified in the articular cartilage. In the superficial zone, the
chondrocytes are elongated, and the collagen fibers are oriented parallel to the articular
surface. In the middle zone or transitional zone, chondrocytes and collagen fibers are
oriented randomly. In the deep zone, the chondrocytes are arranged in columns, and the
collagen fibers are oriented vertical to the articular cartilage. The calcified zone is the
transitional zone between the cartilage and the subchondral bone.
Question 23 of 100
A short statured man standing 4 feet 6 inches with rhizomelic limb shortening has a
father and daughter with a similar short-statured appearance but has a normal-height
mother. In what gene is the mutation most likely to have occurred?

1. SLC26A2
2. COL1A1
3. ACVR1
4. FGFR3
Discussion: 4

The most common form of inherited rhizomelic dwarfism is achondroplasia, caused by a


mutation in the FGFR3 gene. The inheritance pattern is autosomal dominant and is
consistent with the previously described scenarios. COL1A1 mutations are causative for
osteogenesis imperfecta; they are autosomal dominant and recessive, and affected people
can be short statured but not rhizomelic. ACVR1 mutation is associated with
fibrodysplasia ossificans progressive which can be inherited in an autosomal dominant
pattern, but is characterized by severe heterotopic ossification rather than rhizomelic
malformity. Diastrophic dysplasia caused by mutation in SLC26A2 and is transmitted in
autosomal recessive pattern, but is not typically rhizomelic.

Question 24 of 100
When applying the technique shown in Figures 1 and 2, when would the surgeon expect
concentrations of bone morphogenetic protein-2 (BMP-2) or bone promoting proteins to
peak, and when should he or she expect the concentrations to return to baseline?

When applying the technique shown in Figures 1 and 2, when would the surgeon expect
concentrations of bone morphogenetic protein-2 (BMP-2) or bone promoting proteins to
peak, and when should he or she expect the concentrations to return to baseline?

1. 2 weeks, 3 months
2. 2 weeks, 6 months
3. 4 weeks, 3 months
4. 4 weeks, 6 months
Discussion: 4

The surgeons in Figure 2 are attempting to use the induced–membrane technique


described by Masquelet. After the placement of a cement spacer, a well-vascularied
membrane forms around the spacer that harbors proteins such as BMP-2 and vascular
endothelial growth factor that can promote bony healing. Basic science studies and
animal models indicate that peak concentrations of these proteins occur at 4 weeks and
return to baseline by around 6 months.

Question 25 of 100
CLINICAL SITUATION

Figures 1 through 3 display radiographs and an MRI from a 29-year-old man who has
complained of left leg pain since sustaining a gunshot wound 18 months ago. He denies
any fevers or chills but does have pain and drainage from his wound. His erythrocyte
sedimentation rate (ESR) is 105 mm/h (reference range: 0-20 mm/h), C-reactive protein
(CRP) level is 12 mg/L (reference range: 0.08-3.1 mg/L), white blood cell (WBC) count
is 8,000 /µL (reference range: 4500-11000 /µL), and vitamin D level is 15 ng/mL
(reference range: 20-40ng/ml).

What is the tissue indicated by the arrow in Figure 4?

1. An unresorbed butterfly fragment


2. Nonviable, infected bone
3. Heterotopic ossification
4. Reactive bone formation

Discussion: 2
This patient has chronic osteomyelitis and infected nonunion of the tibia. The bone
indicated by the arrow is sequestrum, or infected, dead bone. Involucrum is new bone
laid down around the infection. Treatment of this infection requires debridement and
local antibiotic delivery, often accomplished with an antibiotic-impregnated cement. The
elution characteristics often vary by cement type, but generally, peak concentrations are
reached in 24 hours, and the concentrations can remain bactericidal for 4 months.

Vancomycin is a common antibiotic used in cement delivery. Its mechanism of action


disrupts cell-wall synthesis. It also kills bacteria in a time-dependent manner, not in a
concentration-dependent way like tobramycin and other aminoglycosides. This means
that, for the drug to be effective, the concentration must be above the minimal inhibitory
concentration between doses, illustrating why following a vancomycin trough is
important. Drugs that kill in a concentration-dependent way still have effect, even after a
limited exposure; this fact explains why the dosing for gentamycin is recommended once
per day for open fractures.

Lastly, a biofilm is characterized by bacteria entering a no-growth, or sessile, phase,


which makes them even more resistant to antibiotics that depend on replication to carry
out their effect. Planktonic bacteria are the free bacteria that spread and often cause sepsis
and active infection.

Question 26 of 100LINICAL SITUATION


A 23-year-old man sustained a moderately comminuted femoral shaft fracture and simple
oblique humeral shaft fracture in a motor vehicle collision. The humerus underwent open
reduction and internal fixation with compression plating, whereas the femur was treated
with an intramedullary device.

One determinant of primary versus secondary bone healing is that the response of bone
remodeling is based on the stress applied to it. Which principle governs this process?

1. Hueter-Volkmann law
2. Piezoelectric changes
3. Regeneration potential
4. Wolff law
Discussion : 4

Bone remodeling includes primary and secondary healing. Primary healing occurs with
absolute fixation, minimal or no comminution, and decreased fracture motion; it is
accomplished through Haversian remodeling. This process proceeds through osteoclast
resorption, osteoblastic layering, lamellar layering, and finally, cement line creation. The
Wolff law is defined as bone remodeling in response to mechanical stress. Increased
fracture comminution or motion and relative fixation are associated with secondary
healing and callus formation.

Question 27 of 100
Necrotizing fasciitis that occurs following varicella infection in children most often is
caused by what bacteria?

1. Group A streptococcus
2. Group B streptococcus
3. Staphylococcus aureus
4. Clostridium perfringens

Discussion: 1

Varicella zoster virus infection leads to one of two clinically distinct forms of disease:
varicella (chickenpox) and herpes zoster (shingles). Before a vaccine was introduced in
1995, an estimated four million cases occurred per year in the United States, along with
nearly 11,000 hospital admissions and 100 deaths annually. In children, chickenpox is
generally a mild disease and presents with fever, malaise, pharyngitis, and loss of
appetite, followed by the development of a pruritic vesicular rash. Complications,
including invasive skin and soft-tissue infections, pneumonia, and encephalitis, occur in
around 2% of patients, with skin and soft-tissue infections being most common.
Necrotizing fasciitis is an uncommon but devastating complication and is associated with
a high mortality rate. Early identification, followed by aggressive surgical debridement
and appropriate antibiotic selection is critical. Group A β-hemolytic Streptococcus, most
commonly Streptococcus pyogenes, is the most common causative organism, occurring in
up to 80% of patients in some series. In fact, between 15% and 30% of invasive group A
streptococcal infections are associated with varicella. Group B Streptococcus is a
potential cause of infection in neonates and young infants, especially those having poor
prenatal care. Staphylococcus aureus is the most common cause of musculoskeletal
infections in the general population. Clostridium perfringens is the most common cause
of traumatic clostridial myonecrosis or “gas gangrene” but is not common following
varicella infection.

Question 28 of 100
Figures 1 and 2 show radiographs from a 12-year-old girl in the emergency department.
She reports a 3-week history of left groin pain. The girl denies any antecedent trauma or
constitutional symptoms. She has been able to bear weight and denies any history of
endocrine or renal disorders. On examination, she is moderately obese and has limited
passive motion of the hip secondary to pain. Displacement has occurred primarily
through which zone of the physis?

1. Resting
2. Proliferative
3. Hypertrophic
4. Primary spongiosa

Discussion: 3

The radiographs reveal a left slipped capital femoral epiphysis (SCFE). SCFE leads to
displacement of the metaphysis relative to the epiphysis, with the metaphysis moving
anterior and rotating externally. It is the most common disorder affecting the adolescent
hip. Risk factors include obesity, endocrine disorders, renal osteodystrophy, and male
sex. Displacement occurs primarily through the hypertrophic zone, although some
displacement may occur through the zone of provisional calcification. The resting zone is
usually normal but may account for a smaller percentage of the overall physis thickness
given the relative increase in the thickness of the proliferative and hypertrophic zones.
Question 29 of 100
CLINICAL SITUATION

A 23-year-old man sustained a moderately comminuted femoral shaft fracture and simple
oblique humeral shaft fracture in a motor vehicle collision. The humerus underwent open
reduction and internal fixation with compression plating, whereas the femur was treated
with an intramedullary device.

What factor is most associated with an increased likelihood of primary bone healing?

1. Increased fracture comminution


2. Relative fixation
3. Decreased fracture motion
4. Bridging construct fixation

Discussion: 3

Bone remodeling includes primary and secondary healing. Primary healing occurs with
absolute fixation, minimal or no comminution, and decreased fracture motion; it is
accomplished through Haversian remodeling. This process proceeds through osteoclast
resorption, osteoblastic layering, lamellar layering, and finally, cement line creation. The
Wolff law is defined as bone remodeling in response to mechanical stress. Increased
fracture comminution or motion and relative fixation are associated with secondary
healing and callus formation.

Question 30 of 100
CLINICAL SITUATION

A 23-year-old man sustained a moderately comminuted femoral shaft fracture and simple
oblique humeral shaft fracture in a motor vehicle collision. The humerus underwent open
reduction and internal fixation with compression plating, whereas the femur was treated
with an intramedullary device.

What type of healing is expected after this type of fixation in the femur?

1. Intramembranous ossification
2. Secondary bone healing with Haversian remodeling
3. Secondary bone healing with callus formation
4. Primary bone healing with Haversian remodeling
Discussion : 3

Bone remodeling includes primary and secondary healing. Primary healing occurs with
absolute fixation, minimal or no comminution, and decreased fracture motion; it is
accomplished through Haversian remodeling. This process proceeds through osteoclast
resorption, osteoblastic layering, lamellar layering, and finally, cement line creation. The
Wolff law is defined as bone remodeling in response to mechanical stress. Increased
fracture comminution or motion and relative fixation are associated with secondary
healing and callus formation.

Question 31of 100


Intervertebral disk degeneration results in what changes to the collagen content and
nonenzymatic cross-linking?

1. Increase in collagen II, increase in nonenzymatic cross-linking of the collagen


2. Decrease in collagen II, decrease in nonenzymatic cross-linking of the collagen
3. Increase in collagen I, increase in nonenzymatic cross-linking of the collagen
4. Decrease in collagen I, decrease in nonenzymatic cross-linking of the collagen

Discussion: 3

Question 32 of 100
A 35-year-old man sustains a lower extremity injury during a motor vehicle collision. He
is transported to a local trauma center, where he is diagnosed with an intra-articular
fracture. Concerned about his prognosis and the risk of developing arthritis, he asks the
treating surgeon what type of recovery he may expect after his injury.

Following intra-articular fracture, the risk of posttraumatic arthritis is correlated strongly


with what risk factor?

1. Patient sex
2. Duration of patient tobacco use
3. Intra-articular fracture severity
4. Axial versus torsional mechanisms

Discussion : 2
Posttraumatic arthritis is common, and its prevalence likely is underestimated following
joint injury. Cartilage injuries can include surface defects, chondral fractures, or intra-
articular fractures, which involve the cartilage surface and subchondral bone. Intra-
articular fractures place patients at risk for posttraumatic arthritis because of the
magnitude of the energy imparted to the joint surface and the extent of damage to the
joint. Compared with other joints, the ankle joint is most prone to developing
posttraumatic arthritis. The condition is debilitating and is similar in impact on the quality
of life to end-stage kidney disease and congestive heart failure. Intra-articular fracture
severity is predictive of eventual arthritis development. Following fracture reduction,
joint incongruities increase the contact stress to the remaining joint, contributing to
further joint degeneration. Distraction arthroplasty is an option that does not sacrifice the
patient’s native joint, but it has not been shown to be superior to ankle fusion for end-
stage arthritis of the ankle. Weight bearing or articulated distraction frames offer the
advantage of fluctuating the intra-articular hydrostatic pressure of the synovial fluid,
which is favorable biologically for the repairing cartilage.

Question 33 of 100
Which protein overexpression resulting from a translocation in a subpopulation of cells
within the synovium is associated with pigmented villonodular synovitis (PVNS) or giant
cell tumor of the tendon sheath (GCTTS)?

1. Colony stimulating factor 1 (CSF 1)


2. Receptor activator of nuclear factor kappa B ligand (RANKL)
3. Tumor necrosis factor alpha (TNF-α)
4. Interleukin-2 (IL-2)

Discussion: 1

Several studies have found a subpopulation of cells within PVNS and GCTTS and some
other types of chronic synovial inflammatory conditions that show an increased
expression of CSF 1. This overexpression is thought to result from a translocation within
this cell subpopulation. RANKL is a protein that is released by osteoblasts, which bind to
RANK on osteoclasts, leading to activation of the osteoclasts. This interaction is involved
in the bone resorption frequently seen in PVNS, but the tumor cells themselves are not
known to express RANKL directly, and RANKL is not known to be associated with a
translocation. TNF-α and IL-2 are proinflammatory proteins released by the white blood
cells at the sites of inflammation. Although likely important to the inflammatory process
present in PVNS and GCTTS, TNF-α and IL-2 are not associated with a subpopulation of
cells within the tumor.

Question 34 of 100
CLINICAL SITUATION

A 35-year-old man sustains a lower extremity injury during a motor vehicle collision. He
is transported to a local trauma center, where he is diagnosed with an intra-articular
fracture. Concerned about his prognosis and the risk of developing arthritis, he asks the
treating surgeon what type of recovery he may expect after his injury.

The reparative response following osteochondral fracture, compared with that of chondral
fractures, includes

1. restoration of the normal organization of collagen fibrils.


2. a paucity of inflammatory cells in the fracture gap.
3. hyperproliferation of synovial tissue.
4. formation of a fibrin clot and an inflammatory response.

Discussion : 4

Posttraumatic arthritis is common, and its prevalence likely is underestimated following


joint injury. Cartilage injuries can include surface defects, chondral fractures, or intra-
articular fractures, which involve the cartilage surface and subchondral bone. Intra-
articular fractures place patients at risk for posttraumatic arthritis because of the
magnitude of the energy imparted to the joint surface and the extent of damage to the
joint. Compared with other joints, the ankle joint is most prone to developing
posttraumatic arthritis. The condition is debilitating and is similar in impact on the quality
of life to end-stage kidney disease and congestive heart failure. Intra-articular fracture
severity is predictive of eventual arthritis development. Following fracture reduction,
joint incongruities increase the contact stress to the remaining joint, contributing to
further joint degeneration. Distraction arthroplasty is an option that does not sacrifice the
patient’s native joint, but it has not been shown to be superior to ankle fusion for end-
stage arthritis of the ankle. Weight bearing or articulated distraction frames offer the
advantage of fluctuating the intra-articular hydrostatic pressure of the synovial fluid,
which is favorable biologically for the repairing cartilage.
Question 35 of 100
CLINICAL SITUATION

A 35-year-old man sustains a lower extremity injury during a motor vehicle collision. He
is transported to a local trauma center, where he is diagnosed with an intra-articular
fracture. Concerned about his prognosis and the risk of developing arthritis, he asks the
treating surgeon what type of recovery he may expect after his injury.

Early posttraumatic arthritis develops. To attempt to salvage his native joint, he is


indicated for distraction arthroplasty. A distraction frame that allows partial weight
bearing improves cartilage repair through

1. fluctuations in the intra-articular hydrostatic synovial pressure.


2. maintenance of cartilage stiffness during joint loading.
3. earlier return to a normal postsurgical range of motion.
4. improved maintenance of limb strength, facilitating rehabilitation.

Discussion: 1

Posttraumatic arthritis is common, and its prevalence likely is underestimated following


joint injury. Cartilage injuries can include surface defects, chondral fractures, or intra-
articular fractures, which involve the cartilage surface and subchondral bone. Intra-
articular fractures place patients at risk for posttraumatic arthritis because of the
magnitude of the energy imparted to the joint surface and the extent of damage to the
joint. Compared with other joints, the ankle joint is most prone to developing
posttraumatic arthritis. The condition is debilitating and is similar in impact on the quality
of life to end-stage kidney disease and congestive heart failure. Intra-articular fracture
severity is predictive of eventual arthritis development. Following fracture reduction,
joint incongruities increase the contact stress to the remaining joint, contributing to
further joint degeneration. Distraction arthroplasty is an option that do es not sacrifice the
patient’s native joint, but it has not been shown to be superior to ankle fusion for end-
stage arthritis of the ankle. Weight bearing or articulated distraction frames offer the
advantage of fluctuating the intra-articular hydrostatic pressure of the synovial fluid,
which is favorable biologically for the repairing cartilage.
Question 36 of 100
CLINICAL SITUATION

A 35-year-old man sustains a lower extremity injury during a motor vehicle collision. He
is transported to a local trauma center, where he is diagnosed with an intra-articular
fracture. Concerned about his prognosis and the risk of developing arthritis, he asks the
treating surgeon what type of recovery he may expect after his injury.

Following open reduction and internal fixation, residual incongruity contributes to joint
degeneration through

1. aberrant contact stresses across the joint.


2. collagen formation in the residual fracture gap.
3. a longer required time of prohibited weight bearing.
4. ongoing inflammation generated from the synovium.

Discussion: 1

Posttraumatic arthritis is common, and its prevalence likely is underestimated following


joint injury. Cartilage injuries can include surface defects, chondral fractures, or intra-
articular fractures, which involve the cartilage surface and subchondral bone. Intra-
articular fractures place patients at risk for posttraumatic arthritis because of the
magnitude of the energy imparted to the joint surface and the extent of damage to the
joint. Compared with other joints, the ankle joint is most prone to developing
posttraumatic arthritis. The condition is debilitating and is similar in impact on the quality
of life to end-stage kidney disease and congestive heart failure. Intra-articular fracture
severity is predictive of eventual arthritis development. Following fracture reduction,
joint incongruities increase the contact stress to the remaining joint, contributing to
further joint degeneration. Distraction arthroplasty is an option that does not sacrifice the
patient’s native joint, but it has not been shown to be superior to ankle fusion for end-
stage arthritis of the ankle. Weight bearing or articulated distraction frames offer the
advantage of fluctuating the intra-articular hydrostatic pressure of the synovial fluid,
which is favorable biologically for the repairing cartilage.

Question 37 of 100
Figure 1 shows an MRI from a 54-year-old man who has experienced a substantial
increase in axial back pain that intermittently responds to oral NSAIDs. He works for a
moving company, and his lumbar range of motion has been limited, especially with
flexion. The pathologic change seen on MRI can be attributed to an increase in

1. Type II collagen.
2. proteoglycan.
3. chondrocytic apoptosis.
4. water.

Discussion: 3

Dark disk findings on T2-weighted MRI reveal a reduction in the content of extracellular
water and proteoglycan. Chondrocytes produce and maintain the normal extracellular
matrix. As cells within the disk die, the extracellular matrix declines

Question 38 of 100
How does the proteoglycan content of the nucleus pulposus intervertebral disk change
from the newborn period into adolescence?

1. Chondroitin sulfate and keratan sulfate increase in concentration with age.


2. Chondroitin sulfate and keratan sulfate decrease in concentration with age.
3. Chondroitin sulfate increases and keratan sulfate decreases with age.
4. Chondroitin sulfate decreases and keratan sulfate increases with age.

Discussion: 4

The intervertebral disk is made up of a nucleus pulposus and an anulus fibrosus. The dry
weight of the nucleus pulposus of the intervertebral disk consists mostly of collagen and
proteoglycans. After birth, the side chains of the aggrecans are made mostly of
chondroitin sulfate, which gradually is replaced by keratan sulfate with age.
Question 39 of 100
In posttraumatic arthritis, the initial injury stimulates the production of inflammatory
cytokines. Which cytokine is produced at the highest level on the first day after injury?

1. Interleukin-6 (IL-6)
2. Interleukin-1 beta (IL-1β)
3. Chemokine ligand 22 (CCL-22)
4. Damage-associated molecular patterns (DAMPs)

Discussion: 2

The development of arthritis after joint injury is common and can result from multiple
causes, including cartilage contusion, meniscal injury, ligament tear, or intra-articular
fracture. The accuracy of reduction does not necessarily prevent the development of
posttraumatic arthritis. Data from animal studies of posttraumatic arthritis demonstrate
the production of inflammatory cytokines that lead to chondrocyte death and matrix
destruction. In the first few days after injury IL-1β (predominantly) and tumor necrosis
factor alpha are the primary cytokines produced, followed by nitric oxide, matrix
metalloproteinases, and aggrecanases, which degrade the chondral matrix CCL-22
increases at around 5 days after injury, however. Other factors called DAMPs, which are
generated through the mechanical or enzymatic degradation of joint tissues, also
stimulate an innate inflammatory response.

Question 40 of 100
The anulus fibrosus of the intervertebral disk develops directly from which embryonic
structure?

1. Neural tube
2. Primitive streak
3. Sclerotomal cells
4. Unsegmented notochord

Discussion: 3

Following resegmentation of the sclerotomes, cells from the scleroto mes themselves are
differentiated into anulus fibrosus components.
Question 41 of 100
A 44-year-old man arrives at the trauma bay intubated and sedated following a
motorcycle collision. He has sustained multiple injuries and remains hemodynamically
unstable. His neurologic status is unknown. The trauma team would like to remove the
patient’s cervical orthosis. CT reveals multilevel spondylosis, however, that is
inconclusive for the extent of injury. Which MRI sequence would best show acute
traumatic injury?

1. T2-weighted
2. Gradient echo
3. Short-tau inversion-recovery (STIR)
4. Fluid-attenuated inversion-recovery

Discussion: 3

Determining the extent of injury to the cervical spine in an obtunded polytrauma patient
can be extremely difficult. Complications are encountered, including missed diagnosis or
unnecessarily prolonged cervical collar treatment. STIR sequences reveal early-phase
fluid such as that resulting from contusion and acute trauma. T2-weighted images and T2
fat suppression images are less sensitive to acute-phase fluids because of a lack of fat and
marrow edema suppression.

Question 42 of 100
What is the main function of lubricin in synovial joints?

1. Serves as a component of the extracellular matrix


2. Inhibits matrix metalloproteinase
3. Increases cross-linking between collagen fibers
4. Reduces the coefficient of friction in the joint

Discussion: 4

Lubricin reduces the friction between the surfaces in the joint, leading to decreased shear
forces going through the hyaline cartilage. It is a glycoprotein that is produced by the
chondrocytes in the superficial zone and is not a primary component of the extracellular
matrix. A lack of lubricin has been associated with syndromes causing arthritic changes
at an early age.
Question 43 of 100
Three hours after sustaining a tibial shaft fracture in a motorcycle accident, a 19-year-old
man has increasing pain despite receiving increased narcotic pain medications. The pain
is exacerbated with passive stretching. Paresthesias develop along the tibial and peroneal
nerve distributions. The progression of his conditions results from the compromise of

1. venous outflow relative to arterial inflow.


2. venous outflow independent of arterial inflow.
3. arterial inflow independent of venous outflow.
4. venous outflow and arterial inflow in an independent fashion.

Discussion: 1

Patients with high-energy tibia fractures are at high risk for compartment syndromes.
This patient’s condition displays several key clinical indicators, including increased pain
medication requirements, pain with passive stretching, and paresthesias. The progression
of compartment syndrome results from a compromise of venous outflow relative to
arterial inflow. As this imbalance increases, venous congestion further raises already
elevated compartment pressures and, if the imbalance is untreated, eventually results in
arterial collapse.

Question 44 of 100
Compared with intrinsic tendon repair mechanisms for intrasubstance intrasynovial
flexion tendon tears, extrinsic tendon repair mechanisms have what effects on the speed
of healing and adhesion formation?

1. Faster healing increased adhesion formation


2. Faster healing decreased adhesion formation
3. Slower healing increased adhesion formation
4. Slower healing decreased adhesion formation

Discussion: 1

Intrasynovial tendons are those enclosed within a synovial sheath. Intrinsic repair
mechanisms describe the healing processes that rely on the proliferation and migration of
cells within the injured tendon itself to help bridge the injury. In extrinsic healing
mechanisms, cells from the surrounding tissue outside the tendon invade the damaged
area. Extrinsic healing can represent a substantial part of the overall healing response and
is faster than intrinsic healing for intrasynovial flexor tendons. Extrinsic healing results in
adhesion formation between the tendon and the surrounding tissue, however.

Question 45 of 100
Tendon injury classically occurs during what type of muscle contraction?

1. Concentric
2. Isometric
3. Eccentric
4. Passive stretch

Discussion: 3

Tendons demonstrate both elastic and viscoelastic properties. These biomechanical


properties can be affected by pathologic processes manifesting as tendinosis, weakening
the tendon and predisposing it to rupture. In such circumstances, eccentric muscle
contractions can place maximal stress across the myotendinous junction, resulting in
forced lengthening and rupture.

Question 46 of 100
The immunomodulating medications infliximab and etanercept are used to treat
rheumatoid arthritis (RA) by targeting

1. Receptor activator of nuclear factor kappa-B ligand (RANKL).


2. Interleukin-1 (IL-1).
3. Beta cells.
4. Tumor necrosis factor alpha (TNF-α).

Discussion: 4

Infliximab and etanercept are recombinant drugs designed to inhibit TNF-α, which is a
macrophage-derived cytokine that mediates the destructive inflammatory response of RA.
Anakinra is a biologic agent that targets IL-1. These agents are used in the treatment of
RA. Denosumab is a monoclonal antibody that targets RANKL and is used in the
treatment of osteoporosis
Question 47 of 100
Following Achilles tendon repair, a 33-year-old man is lost to follow-up and becomes
immobilized for 6 months. Concerns about weakness in his tendon from a lack of motion
primarily reflect what fundamental deficiency in this patient’s tendon?

1. Collagen organization
2. Collagen production
3. Paratenon proliferation
4. Paratenon regeneration

Discussion: 1

Some of the fundamental changes associated with the healing of collagen-based tissues
are the organization, reorganization, and remodeling of the collagen fibers. These
processes can be induced and influenced by the application of forces such as directional
tension through motion. Motion may also have some influence on collagen production,
but the most important effect of motion is organization of the collagen along lines of
tension. Also, the paratenon can be a source of support for the healing tendon, but its
proliferation and regeneration are not functions of motion as it relates to tendon healing.

Question 48 of 100
After the repair of an intrasynovial flexor tendon laceration, increasing the mechanical
load during rehabilitation has what type of effect on tendon healing and repair site
gapping?

1. Promotes tendon healing, increases the risk of repair site gapping


2. Promotes tendon healing, reduces the risk of repair site gapping
3. Inhibits tendon healing, increases the risk of repair site gapping
4. Inhibits tendon healing, reduces the risk of repair site gapping

Discussion: 1

Studies have shown that tendon fibroblasts respond positively to mechanical load, and
some amount of mechanical loading is incorporated into many tendon repair
rehabilitation programs. Increased loads run the risk of increasing the repair gap or even
disrupting the repair altogether, however.
Question 49 of 100
Regarding the modulus of elasticity and wear rates, how does cobalt chrome compare
with titanium? Cobalt chrome has a

1. lower modulus of elasticity and increased wear rates.


2. lower modulus of elasticity and reduced wear rates.
3. higher modulus of elasticity and reduced wear rates.
4. higher module of elasticity and increased wear rates.

Discussion:3

Titanium and cobalt chrome are used commonly in orthopaedic implants. Cobalt chrome
has a higher modulus of elasticity and reduced wear rates compared with titanium.
Because of the higher modulus of elasticity, cobalt chrome causes more stress shielding.
Titanium has better biocompatibility, but its wear properties are inferior to those of cobalt
chrome. For this reason, titanium is used mostly in the nonarticular part of orthopaedic
implants.

Question 50 of 100
A 13-year-old girl with progressive adolescent onset idiopathic scoliosis undergoes
corrective surgery with posterior fusion and instrumentation from T2 -12. During the
procedure, several maneuvers are performed to correct a single main thoracic curve.
Which maneuver is responsible for improving apical translation?

1. Direct vertebral rotation


2. Concave distraction
3. Convex compression
4. 90° rod rotation

Discussion: 4

Scoliosis is a three-dimensional deformity of the spine that is multifactorial in etiology.


Halting progression by achieving fusion is the primary aim of surgical intervention, but
safe correction with instrumentation maneuvers aims to improve each of the planes of
deformity. Rod rotation at the apex assists in coronal translation. Distraction and
compression affect both the coronal and sagittal planes. Direct vertebral rotation
maneuvers address vertebral rotation.
Question 51 of 100
Polylactic acid and polyglycolic acid are used to make resorbable orthopaedic implants.
How do the resorption time and stiffness of polylactic acid compare with those of
polyglycolic acid?

1. Polylactic acid has higher stiffness and takes longer to resorb


2. Polyglycolic acid has higher stiffness and takes longer to resorb
3. Polylactic acid has higher stiffness but resorbs faster
4. Polyglycolic acid has higher stiffness but resorbs faster

Discussion: 4

Polylactic and polyglycolic acids are degraded in the body by hydrolysis. Polyglycolic
acid has a modulus of elasticity of 7 GPa and degrades within 6 to 12 months, whereas
polylactic acid has a modulus of elasticity of 2.7 GPa and requires more than 24 months
to resorb fully.

Question52 of 100
Increasing the mineral-to-matrix ratio of bone beyond normal would have what effect on
the modulus of elasticity and the duration of the plastic phase?

1. Increase the modulus of elasticity, shorten the plastic phase


2. Increase the modulus of elasticity, lengthen the plastic phase
3. Decrease the modulus of elasticity, shorten the plastic phase
4. Decrease the modulus of elasticity, lengthen the plastic phase

Discussion: 1

Increasing the mineral content of bone will increase the stiffness and modulus of
elasticity of bone, because an applied fore creates less deformation, and stress creates less
strain. As the increased mineral content increases the stiffness and decreases compliance,
the plastic phase will shorten, leading to a quick transition from the elastic through the
plastic phases and catastrophic failure.
Question 55 of 100
Commonly used to produce orthopaedic implants, 316L stainless steel is an alloy of what
elements?

1. Cobalt, chromium, molybdenum


2. Iron, carbon, chromium, nickel, molybdenum, manganese
3. Iron, carbon, titanium, nickel, molybdenum, manganese
4. Titanium, aluminum, vanadium, iron, oxygen

Discussion:2

316L stainless steel is an alloy of iron, carbon, chromium, nickel, molybdenum, and
manganese, as well as smaller amounts of phosphorus, sulfur, silicon, and nitrogen. Iron
makes up the bulk of stainless steel, and carbon is limited to less than 0.03% to limit
intragranular corrosion. Chromium increases hardness and corrosion resistance.
Likewise, nickel and molybdenum help prevent corrosion.

Cobalt chrome alloy, commonly used in spinal implants, is composed of cobalt,


chromium, and molybdenum. Ti6Al4V titanium alloy is one of the most commonly used
titanium alloys and is composed of titanium, aluminum, vanadium, iron, and oxygen.

Question 56 of 100
The surface wear that occurs between two implant surfaces designed to be in contact with
one another is known as

1. fatigue wear.
2. corrosion wear.
3. third-body wear.
4. adhesion wear.

Discussion: 4

Adhesion wear occurs between implant surfaces that are designed to be in contact.
Corrosion often occurs when contact occurs between different metals, as in a cobalt -
chrome alloy metal head on a titanium taper. Third-body wear is a type of abrasive wear
that occurs when particulates that are usually extra-articular become entrapped in an
articulation and cause wear at the articulating surfaces. Fatigue wear develops from
progressive mechanical use and is the result of repetitive cyclic loads.

Question 57 of 100
Fretting corrosion is defined as

1. motion between two pieces of metal that are loose.


2. the microisolation of oxygen, leading to disruption of passivation.
3. an electrochemical circuit formed between two different metals.
4. micromotion between two pieces of metal that are not loose.

Discussion: 4

Modular junctions are susceptible to fretting corrosion. During fretting corrosion, surface
pressure between contacting surfaces at modular articulations cause friction. Surface
pressure often is initiated by micromotion at the interfaces during mechanical loading.
This micromotion physically can disrupt the passivation layer, leading to the release of
metal particles and the facilitation of crevice corrosion. Micromotion is not the re sult of a
loose implant. Rather, when the joint is loaded, the various components of the modular
implant flex (slip region) while remaining together (stick region)

Question 58 of 100
Crevice corrosion is defined as

1. motion between two pieces of metal that are loose.


2. the microisolation of oxygen, leading to disruption of passivation.
3. an electrochemical circuit formed between two different metals.
4. micromotion between two pieces of metal that are not loose.

Discussion: 2

All modular junctions are susceptible to crevice corrosion. At modular junctions


machined with tight tolerances such as the head-neck junction, the space between the two
surfaces is shielded partially from the aqueous chemical environment of the body fluid.
This microisolation of the tiny gaps (or crevices) between the two surfaces of the implant
is the source of crevice corrosion. The imbalance of oxygen between the cathode (the
surface of the implant outside the crevice) and the anode inside the crevice) creates a
current, even if each part of the modular junction is made from the same metal. At the
anode, oxygen is low, whereas the concentrations of metal, hydrogen, and chloride ions
are high. In this oxygen-poor environment, the surface of the implant cannot repassivate.
If the metal ion released is chromium, it reacts with phosphate to form a black or green,
tarry or glassy precipitate (ie, chromium (III) phosphate and/or other metal oxides) near
the crevice and on the surface of the implant. Hence, the serum chromium level remains
normal or only slightly elevated in patients with corrosion at modular junctions, whereas
the serum cobalt level rises because of its increased solubility. This fact can be used to
distinguish a failed metal-on-metal articulation—which in general has an equivalent rise
in serum cobalt and chromium levels—from a failed modular junction—which in general
has a differential elevation of the serum cobalt level severalfold above that of the serum
chromium level.

Question 59 of 100
What are the effects of remelting and annealing on crystal formation in ultrahigh-
molecular-weight polyethylene (UHMWPE)?

1. Annealing increases and remelting decreases crystal formation


2. Both increase crystal formation
3. Annealing decreases and remelting increases crystal formation
4. Both decrease crystal formation

Discussion: 1

Annealing and remelting are performed after gamma radiation of the UHMWPE to
reduce free radicals. In annealing, the material is heated below the melting point, whereas
in remelting, the material is heated above the melting point. Annealing increases the
crystal formation of the UHMWPE, and remelting decreases crystal formation. Both
processes reduce but do not eradicate free radicals completely.

Question 60 of 100
CLINICAL SITUATION

A surgeon devises an experiment to test the axial loading and rotational stability of bridge
plate fixation. Figure 1 shows a locking compression plate fixed to a forearm simulation
bone with a 2-mm gap between the fragments to simulate comminution.
Placing screws into which holes would provide maximum rigidity to this construct in
axial compression?

1. 1, 2 ,3
2. 1, 2, 5
3. 1, 4, 5
4. 2, 4, 5

Discussion: 2

The model shown in Figure 1 demonstrates a bridge plate concept that attempts to
achieve secondary healing. Strain theory predicts that flexibility of the plate is important
to stimulate healing; however, this flexibility must be balanced with appropriate stability.
Multiple studies have investigated the stress, strain, and flexibility of plate fixation. The
screws nearest the fracture absorb the highest stress in bridge plating, unlike the
application of rigid fixation with direct compression of the bone ends, in which the stress
is maximal at the screws farthest from the fracture.

The rigidity of a construct can be manipulated by changing the working length of the
plate. Creating a longer working length by leaving the screw holes closer to the fracture
site empty reduces the rigidity of the construct. In the second item, two screws close to
the fracture and then one screw far away on either side creates the most rigid construct,
much like an external fixator. Remembering that strain is (change in length)/length, the
surgeon could reduce the strain seen in the implant by increasing the working length of
the plate or by placing screws farther away from the fracture.

Torsional stability is different from bending or axial compression stability. Torsional


stability depends more on the number of screws on either side of the fracture than on the
location of the screws. Biomechanical studies have shown that adding more than four
screws will not improve torsional stability statistically, provided good purchase is
achieved within the bone.
Lastly, as described previously, the working length of the plate is the distance between
the innermost screws spanning the fracture site.

Question 61 of 100
CLINICAL SITUATION

A surgeon devises an experiment to test the axial loading and rotational stability of bridge
plate fixation. Figure 1 shows a locking compression plate fixed to a forearm simulation
bone with a 2-mm gap between the fragments to simulate comminution.

Assuming that screws were placed in all holes, which screw would be expected to
experience the highest stress during axial loading?

1. Screw 1
2. Screw 2
3. Screw 4
4. Screw 5

Discussion: 1

The model shown in Figure 1 demonstrates a bridge plate concept that attempts to
achieve secondary healing. Strain theory predicts that flexibility of the plate is important
to stimulate healing; however, this flexibility must be balanced with appropriate stability.
Multiple studies have investigated the stress, strain, and flexibility of plate fixation. The
screws nearest the fracture absorb the highest stress in bridge plating, unlike the
application of rigid fixation with direct compression of the bone ends, in which the stress
is maximal at the screws farthest from the fracture.

The rigidity of a construct can be manipulated by changing the working length of the
plate. Creating a longer working length by leaving the screw holes closer to the fracture
site empty reduces the rigidity of the construct. In the second item, two screws close to
the fracture and then one screw far away on either side creates the most rigid construct,
much like an external fixator. Remembering that strain is (change in length)/length, the
surgeon could reduce the strain seen in the implant by increasing the working length of
the plate or by placing screws farther away from the fracture.

Torsional stability is different from bending or axial compression stability. Torsional


stability depends more on the number of screws on either side of the fracture than on the
location of the screws. Biomechanical studies have shown that adding more than four
screws will not improve torsional stability statistically, provided good purchase is
achieved within the bone.

Lastly, as described previously, the working length of the plate is the distance between
the innermost screws spanning the fracture site

.
Question 62 of 100
CLINICAL SITUATION

A surgeon devises an experiment to test the axial loading and rotational stability of bridge
plate fixation. Figure 1 shows a locking compression plate fixed to a forearm simulation
bone with a 2-mm gap between the fragments to simulate comminution.

Screws have been placed into holes 2, 3, and 5 on either side of the osteotomy, but the
surgeon wants to make sure that the construct achieves maximal torsional stability, while
avoiding the placement of unnecessary screws. How can the surgeon adjust the fixation to
meet these goals?

1. Move the innermost screw to hole 1 on both sides of the osteotomy.


2. Keep the construct as it is, because it meets these goals.
3. Place an additional screw into hole 1 on both sides of the plate.
4. Place two additional screws into holes 1 and 4 on both sides of the plate.

Discussion: 3

The model shown in Figure 1 demonstrates a bridge plate concept that attempts to
achieve secondary healing. Strain theory predicts that flexibility of the plate is important
to stimulate healing; however, this flexibility must be balanced with appropriate stability.
Multiple studies have investigated the stress, strain, and flexibility of plate fixation. The
screws nearest the fracture absorb the highest stress in bridge plating, unlike the
application of rigid fixation with direct compression of the bone ends, in which the stress
is maximal at the screws farthest from the fracture.

The rigidity of a construct can be manipulated by changing the working length of the
plate. Creating a longer working length by leaving the screw holes closer to the fracture
site empty reduces the rigidity of the construct. In the second item, two screws close to
the fracture and then one screw far away on either side creates the most rigid construct,
much like an external fixator. Remembering that strain is (change in length)/length, the
surgeon could reduce the strain seen in the implant by increasing the working length of
the plate or by placing screws farther away from the fracture.

Torsional stability is different from bending or axial compression stability. Torsional


stability depends more on the number of screws on either side of the fracture than on the
location of the screws. Biomechanical studies have shown that adding more than four
screws will not improve torsional stability statistically, provided good purchase is
achieved within the bone. Lastly, as described previously, the working length of the plate
is the distance between the innermost screws spanning the fracture site.

Question 63 of 100
CLINICAL SITUATION

A surgeon devises an experiment to test the axial loading and rotational stability of bridge
plate fixation. Figure 1 shows a locking compression plate fixed to a forearm simulation
bone with a 2-mm gap between the fragments to simulate comminution.

Assuming that screws have been placed into holes 2, 3, and 5, what is considered to be
the working length of the plate?

1. The distance between the screw holes labeled “2”


2. The distance between the screw holes labeled “5”
3. The distance from the center of the fracture (or plate) to the screw hole labeled “2”
4. The distance from the center of the fracture (or plate) to the screw hole labeled “5 ”

Discussion : 1

The model shown in Figure 1 demonstrates a bridge plate concept that attempts to
achieve secondary healing. Strain theory predicts that flexibility of the plate is important
to stimulate healing; however, this flexibility must be balanced with appropriate stability.
Multiple studies have investigated the stress, strain, and flexibility of plate fixation. The
screws nearest the fracture absorb the highest stress in bridge plating, unlike the
application of rigid fixation with direct compression of the bone ends, in which the stress
is maximal at the screws farthest from the fracture.

The rigidity of a construct can be manipulated by changing the working length of the
plate. Creating a longer working length by leaving the screw holes closer to the fracture
site empty reduces the rigidity of the construct. In the second item, two screws close to
the fracture and then one screw far away on either side creates the most rigid construct,
much like an external fixator. Remembering that strain is (change in length)/length, the
surgeon could reduce the strain seen in the implant by increasing the working length of
the plate or by placing screws farther away from the fracture.

Torsional stability is different from bending or axial compression stability. Torsional


stability depends more on the number of screws on either side of the fracture than on the
location of the screws. Biomechanical studies have shown that adding more than four
screws will not improve torsional stability statistically, provided goo d purchase is
achieved within the bone. Lastly, as described previously, the working length of the plate
is the distance between the innermost screws spanning the fracture site.

Question 64 of 100
Increased cross-linking of ultrahigh-molecular-weight polyethylene (UHMWPE) results
in

1. decreased wear and increased toughness.


2. decreased wear and decreased toughness.
3. increased wear and decreased toughness.
4. increased wear and increased toughness.

Discussion: 2

Studies have shown statistically significantly decreased wear rates in joint arthroplasty
when highly cross-linked UHMWPE is used in place of conventional polyethylene. These
results have been impressive, but the decreased toughness of highly cross-linked
UHMWPE has been associated with such complications as rim fractures found in
retrieved acetabular components.

Question 65 of 100
CLINICAL SITUATION

A study evaluating hip arthroscopy for labral tears enrolls more than 200 participants,
who are randomized into two groups: surgical and nonsurgical care which included
physical therapy and NSAIDs. The primary endpoint was a composite consisting of: 1)
the patient reporting change between preintervention pain as reported on the 10 -point
visual analog scale and the postintervention score at 2 years, and 2) absence of major
adverse events, including death, return to the operating room, and major neurologic
injury.

Which party in the study could be blinded to reduce some potential bias?

1. Surgeon
2. Patients
3. Data and Safety Monitoring Board
4. Data analyst

Discussion: 4

The goal of this series of questions is to test the knowledge of study design. Poolman and
associates have described four parties who can be subject to blinding, and it is important
to clearly state in the study methodology who is blinded because of the various types
of bias. In this case, because of the nature of the study design, only the data analysts are
able to be blinded practically. Allocation concealment is an aspect of study design that
can be applied to any clinical trial. It is important to prevent the treating physician from
being able to guess the group to which the next subject will be randomized. Otherwise, an
inherent risk of selection bias is present. All the methods are predictable except
randomization with opaque envelopes. Physician-driven outcomes are a concern in
research. In such instances, the physician conducting the study also determines when the
variables he or she is measuring are met. This practice leads to obvious clinical bias. One
way of helping to reduce such bias is to set strict, a priori definitions of treatment success
and failure, leaving less room for subjectivity.Currently, the handling of conflicts of
interest is a challenging problem. Although some groups support stricter options, the
existing standard is for the physician to openly disclose any real or potential conflicts of
interest.

Question 66 of 100
CLINICAL SITUATION

A study evaluating hip arthroscopy for labral tears enrolls more than 200 participants,
who are randomized into two groups: surgical and nonsurgical care which included
physical therapy and NSAIDs. The primary endpoint was a composite consisting of: 1)
the patient reporting change between preintervention pain as reported on the 10 -point
visual analog scale and the postintervention score at 2 years, and 2) absence of major
adverse events, including death, return to the operating room, and major neurologic
injury.

What would be the most reasonable study design option to limit the potential bias
inherent in the subjectivity of choosing when to return to the operating room?

1. Requiring a second opinion by an independent surgeon before returning to the


operating room
2. Recording the factors that led treating surgeons to decide to return to the operating
room
3. Stipulating that a committee of treating surgeons must adjudicate all cases
possibly needing a return to the operating room
4. Setting strict a priori definitions for the need to return to the operating room

Discussion: 4

The goal of this series of questions is to test the knowledge of study design. Poolman and
associates have described four parties who can be subject to blinding, and it is important
to clearly state in the study methodology who is blinded because of the various types
of bias. In this case, because of the nature of the study design, only the data analysts are
able to be blinded practically.
Allocation concealment is an aspect of study design that can be applied to any clinical
trial. It is important to prevent the treating physician from being able to guess the group
to which the next subject will be randomized. Otherwise, an inherent risk of selection
bias is present. All the methods are predictable except randomization with opaque
envelopes.

Physician-driven outcomes are a concern in research. In such instances, the physician


conducting the study also determines when the variables he or she is measuring are met.
This practice leads to obvious clinical bias. One way of helping to reduce such bias is to
set strict, a priori definitions of treatment success and failure, leaving less room for
subjectivity.

Currently, the handling of conflicts of interest is a challenging problem. Although some


groups support stricter options, the existing standard is for the physician to openly
disclose any real or potential conflicts of interest.

Question 67 of 100
A recent nonrandomized prospective study was completed evaluating the efficacy of
bracing in patients with adolescent idiopathic scoliosis. The outcome measurements were
progression and need for posterior instrumentation and fusion. In the highly compliant
group (brace wear of more than 10 hours per day), two of 31 patients (6%) required
surgery. In the noncompliant group, 12 of 27 patients (44%) required surgery. Of patients
who were considered to be highly compliant, how many needed to be treated with
bracing to avoid one surgery?

1. 2
2. 3
3. 7
4. 38

Discussion : 3

The statistic number needed to treat (NNT) is the number of patients who must be treated
with a particular intervention, in this case bracing, in order to achieve one additional
favorable outcome, in this case avoidance of surgery. It is defined as 1/absolute risk
reduction (ARR). In this question, the risk of needing surgery is 6% in the highly
compliant group and 44% in the noncompliant group. Thus, the ARR is 38% or .38, and
the NNT is 2.6, which is rounded up, yielding an NNT of 3.

Question 68 of 100
CLINICAL SITUATION

Figures 1 and 2 display the radiographs from a 16-year-old boy with Lesch-Nyhan
syndrome. The boy has severe mental retardation, displays self-mutilating behavior, and
is wheelchair bound. He has progressive scoliosis. The caregiver reports a great deal of
difficulty positioning him in the wheelchair as well as difficulty with hygiene.

This syndrome displays which inheritance pattern?

1. Autosomal dominant
2. Autosomal recessive
3. X-linked dominant
4. X-linked recessive

Discussion: 4

Lesch-Nyhan syndrome is a rare X-linked recessive disorder found almost exclusively in


males. It is characterized most notably by self-mutilation and aggressive behavior.
Additionally, patients typically have severe mental retardation and may exhibit
choreoathetosis. The underlying etiology is an absence of HPRT. This enzyme is
involved in purine metabolism, and affected individuals have considerably elevated uric
acid. Self-mutilation frequently involves biting of the lips and fingers; dental extractions
frequently are needed. Hip dysplasia, including subluxation and dislocation, is very
common in patients with Lesch-Nyhan syndrome, occurring in 66% of individuals in one
study. Clinical manifestations of gout also are seen frequently. Scoliosis and other spinal
problems as well as infections related to self-mutilation also may be seen.

A deficiency of N-acetylgalactosamine-6-sulfate sulfatase is seen in Morquio syndrome.


A deficiency of alpha-L-iduronidase is observed in Hurler syndrome. A deficiency of
beta-glucocerebrosidase in seen in Gaucher disease. Congenital radial head dislocation
may present in isolation or may be seen with Nail-patella syndrome or Cornelia de Lange
syndrome. Congenital pseudarthrosis of the tibia (CPT) frequently is associated with
neurofibromatosis (NF) type 1, and approximately 50% of those with CPT have NF.
Equinovarus foot deformity can be observed in multiple neuromuscular syndromes,
including cerebral palsy and myelomeningocele, but is seen less commonly in patients
with Lesch-Nyhan syndrome.

Question 69 of 100
The overexpression of which factor is responsible for dilatation of the aorta in patients
with mutations in the fibrillin gene?

1. Bone morphogenetic protein-2


2. Indian hedgehog (IHH)
3. Transforming growth factor beta (TGF-β)
4. Osteocalcin

Discussion: 3

Fibrillin gene mutation is seen in patients with Marfan syndrome, which prevents TGF-β
binding, resulting in the increase of TGF-β levels in tissue. This increase has been
associated with aortic dilatation. IHH is responsible for modulation of the growth and
proliferation of chondrocytes in the growth plate. Osteocalcin is a protein produced by
osteoblasts
Question 70 of 100
A pharmaceutical company is evaluating the efficacy of a biologic agent to enhance
fusion rates in lumbar arthrodesis procedures. For statistical analysis, a large number of
patients are separated into two cohorts depending on fusion status. To determine a
statistical numerical difference between the two groups, which test should be employed?

1. Chi-square
2. Fisher exact
3. Mann-Whitney
4. Unpaired t

Discussion: 1

Patients in the study are separated into fusion and nonfusion groups by a categorical
variable. The Fischer exact test compares categorical data in smaller distinct numbers. T-
tests compare continuous data, unpaired for different individuals and paired for matched
samples. The chi-square test compares categorical data in larger formats.

Question 71 of 100
CLINICAL SITUATION

A study evaluating hip arthroscopy for labral tears enrolls more than 200 participants,
who are randomized into two groups: surgical and nonsurgical care which included
physical therapy and NSAIDs. The primary endpoint was a composite consisting of: 1)
the patient reporting change between preintervention pain as reported on the 10-point
visual analog scale and the postintervention score at 2 years, and 2) absence of major
adverse events, including death, return to the operating room, and major neurologic
injury.

What method of allocation would best limit selection bias?

1. Fixed block randomization with block sizes of two


2. Randomization of patients according to the day of the week
3. Simple randomization with assignments placed in an opaque envelope
4. Strict alternation of subject assignments between groups

Discussion: 3
The goal of this series of questions is to test the knowledge of study design. Poolman and
associates have described four parties who can be subject to blinding, and it is important
to clearly state in the study methodology who is blinded because of the various types
of bias. In this case, because of the nature of the study design, only the data analysts are
able to be blinded practically.

Allocation concealment is an aspect of study design that can be applied to any clinical
trial. It is important to prevent the treating physician from being able to guess the group
to which the next subject will be randomized. Otherwise, an inherent risk of selection
bias is present. All the methods are predictable except randomization with opaque
envelopes.

Physician-driven outcomes are a concern in research. In such instances, the physician


conducting the study also determines when the variables he or she is measuring are met.
This practice leads to obvious clinical bias. One way of helping to reduce such bias is to
set strict, a priori definitions of treatment success and failure, leaving less room for
subjectivity.

Currently, the handling of conflicts of interest is a challenging problem. Although some


groups support stricter options, the existing standard is for the physician to openly
disclose any real or potential conflicts of interest.

Question 72 of 100
The basement membrane and the interstitial stroma are the foremost obstacles to cancer
cell invasion. What enzymes have been shown to be vital for tumor invasion?

1. Matrix metalloproteinases (MMPs)


2. Alkaline phosphatases
3. Tartrate resistant acid phosphatases
4. Cathepsins

Discussion: 1

Proteolytic enzymes (proteinases) are divided into two major groups: metalloproteinases
(collagenase, gelatinase, and stromelysin) and cathepsins B and D. The family of
enzymes that has been shown to be vital for matrix destruction during tumor invasion is
the MMP family. The MMPs comprise a family of zinc-binding enzymes that degrade
components of the extracellular matrix. MMPs can be secreted directly by tumor cells or
by normal host cells that are stimulated by tumor-secreted cytokines. The secretion of
MMPs has been correlated with tumor invasiveness, and metalloproteinase inhibitors
have been shown to inhibit the invasiveness of normal and malignant cells

Question 73 of 100
What study is used to statistically combine multiple similar studies and derive a pooled
estimate closest to the truth of a particular research question?

1. Meta-analysis
2. Systematic review
3. Cohort study
4. Case-control study

Discussion: 1

A systematic review evaluates a clearly formulated question and uses systematic and
explicit methods to identify, select, and critically appraise relevant research and to
collect and analyze data from the studies that are included in the review. Statistical
methods (meta-analysis) may or may not be used to analyze and summarize the results
of the included studies. Meta-analysis refers to the use of statistical techniques in a
systematic review to integrate the results of included studies. A cohort study is best used
to evaluate the effects of risk factors on the outcome of a disease process or treatment. A
case-control study evaluates the outcome of a disease process or treatment and compares
a group of patients that have the disease process with a group that does not have it

Question 74 of 100
A surgeon has decided to implement a new draping technique for the open reduction and
internal fixation of ankle fractures that she believes will reduce waste. All materials have
received clearance from the Food and Drug Administration. She has completed 20 such
cases, and now wishes to retrospectively review this case series to perform a cost -savings
analysis, the results of which will be presented at the Hospital Quality Control Committee
Meeting. She has no plans to publish her data in peer reviewed journals. This type of
investigation can be described as
1. exempt human subjects research
2. expedited human subjects research
3. human subjects research requiring full board
4. process improvement

Discussion: 4

The primary purpose of the institutional review board (IRB) is to ensure the privacy and
safety of human research subjects. The IRB offers three different levels of review
depending on the risk level of the study, and the full board is reserved for investigations
involving the highest risk. Studies involving deidentified data and those that present no
more than minimal risk can be considered exempt from review. In this case, however, the
surgeon simply wishes to perform a retrospective review of her own cases for quality
improvement. Therefore, this type of review is not research and is considered process
improvement.

Question 75 of 100
Figure 1 shows a histology slide from a patient’s bone biopsy. This patient is most likely
to have what genetic abnormality?
Figure 1 used with permission from Frassica FJ. Metabolic bone and inflammatory joint disease. In: Lieberman JR,
ed. AAOS Comprehensive Orthopaedic Review. Vol 1. Rosemont, IL: American Academy of Orthopaedic Surgeons;
2009:491-502, Figure 2B.

1. 11;22 chromosomal translocation


2. 5q35-qter (ubiquitin-binding protein sequestosome-1) mutation
3. Fibroblast growth factor receptor 3 (FGFR3) mutation
4. STY-SSX1 or STY-SSX2 fusion

Discussion: 2

This hematoxylin and eosin stain shows the disorganized bone and multiple cement lines
typical of Paget disease. Paget disease has been linked to mutations in a sequestosome
gene, which has multiple cellular activities. The mechanism by which mutation results in
the Paget phenotype is not completely understood. FGFR3 mutation is associated with
achondroplasia, which would display a normal appearing bone histology. 11;22
translocation is associated with Ewing sarcoma, and a related biopsy would display
multiple small round blue cells. The X;18 translocation causing fusion of the STY and
SSX genes is associated with synovial sarcoma, which would appear on biopsy as bone
with hypercellular soft tissue.

Question 76 of 100
CLINICAL SITUATION

Figures 1 and 2 display the radiographs from a 16-year-old boy with Lesch-Nyhan
syndrome. The boy has severe mental retardation, displays self-mutilating behavior, and
is wheelchair bound. He has progressive scoliosis. The caregiver reports a great deal of
difficulty positioning him in the wheelchair as well as difficulty with hygiene.

This condition is caused by a deficiency of which protein?

1. N-acetylgalactosamine-6-sulfate sulfatase
2. Alpha-L-iduronidase
3. Hypoxanthine-guanine phosphoribosyltransferase (HRPT)
4. Beta-glucocerebrosidase

Discussion: 3
Lesch-Nyhan syndrome is a rare X-linked recessive disorder found almost exclusively in
males. It is characterized most notably by self-mutilation and aggressive behavior.

Additionally, patients typically have severe mental retardation and may exhibit
choreoathetosis. The underlying etiology is an absence of HPRT. This enzyme is
involved in purine metabolism, and affected individuals have considerably elevated uric
acid. Self-mutilation frequently involves biting of the lips and fingers; dental extractions
frequently are needed. Hip dysplasia, including subluxation and dislocation, is very
common in patients with Lesch-Nyhan syndrome, occurring in 66% of individuals in one
study. Clinical manifestations of gout also are seen frequently. Scoliosis and other spinal
problems as well as infections related to self-mutilation also may be seen.

A deficiency of N-acetylgalactosamine-6-sulfate sulfatase is seen in Morquio syndrome.


A deficiency of alpha-L-iduronidase is observed in Hurler syndrome. A deficiency of
beta-glucocerebrosidase in seen in Gaucher disease. Congenital radial head dislocation
may present in isolation or may be seen with Nail-patella syndrome or Cornelia de Lange
syndrome. Congenital pseudarthrosis of the tibia (CPT) frequently is associated with
neurofibromatosis (NF) type 1, and approximately 50% of those with CPT have NF.
Equinovarus foot deformity can be observed in multiple neuromuscular syndromes,
including cerebral palsy and myelomeningocele, but is seen less commonly in patients
with Lesch-Nyhan syndrome.

Question 77 of 100
You are serving as the principal investigator (PI) of a site that is taking part in a
multicenter randomized clinical trial. Your site will enroll 10 patients within the study
that includes more than 1,000 patients. So far you have enrolled eight patients, four in
each of two treatment groups, but you have noticed that two in one group stayed in the
hospital much longer than the others. What entity should investigate whether this
occurrence is a concern in the study overall?

1. You, as the site PI


2. The research coordinator at the primary investigative site
3. Your institutional review board (IRB)
4. The Data and Safety Monitoring Board (DSMB)
Discussion: 4

The DSMB is a group that serves as part of the oversight process of a clinical trial to
independently monitor and assess the conduction of a study. Its duties include the
monitoring of events that may be considered adverse outcomes and the review of safety
issues for the study. It is the duty of the PI to report adverse events and to act on them
when needed, but neither the PI at the site, the site’s IRB, nor the research coordinator
have access to all of the data and all lack the independence needed to investigate overall
trends.

Question 78 of 100
CLINICAL SITUATION

Figures 1 and 2 display the radiographs from a 16-year-old boy with Lesch-Nyhan
syndrome. The boy has severe mental retardation, displays self-mutilating behavior, and
is wheelchair bound. He has progressive scoliosis. The caregiver reports a great deal of
difficulty positioning him in the wheelchair as well as difficulty with hygiene.

Laboratory studies reveal elevated serum

1. creatine kinase.
2. uric acid.
3. alkaline phosphatase.
4. beta-hexosaminidase.
Discussion: 2

Lesch-Nyhan syndrome is a rare X-linked recessive disorder found almost exclusively in


males. It is characterized most notably by self-mutilation and aggressive behavior.
Additionally, patients typically have severe mental retardation and may exhibit
choreoathetosis. The underlying etiology is an absence of HPRT. This enzyme is
involved in purine metabolism, and affected individuals have considerably elevated uric
acid. Self-mutilation frequently involves biting of the lips and fingers; dental extractions
frequently are needed. Hip dysplasia, including subluxation and dislocation, is very
common in patients with Lesch-Nyhan syndrome, occurring in 66% of individuals in one
study. Clinical manifestations of gout also are seen frequently. Scoliosis and other spinal
problems as well as infections related to self-mutilation also may be seen.

A deficiency of N-acetylgalactosamine-6-sulfate sulfatase is seen in Morquio syndrome.


A deficiency of alpha-L-iduronidase is observed in Hurler syndrome. A deficiency of
beta-glucocerebrosidase in seen in Gaucher disease. Congenital radial head dislocation
may present in isolation or may be seen with Nail-patella syndrome or Cornelia de Lange
syndrome. Congenital pseudarthrosis of the tibia (CPT) frequently is associated with
neurofibromatosis (NF) type 1, and approximately 50% of those with CPT have NF.
Equinovarus foot deformity can be observed in multiple neuromuscular syndromes,
including cerebral palsy and myelomeningocele, but is seen less commonly in patients
with Lesch-Nyhan syndrome.

Question 79 of 100
Your colleagues have identified an immunologic syndrome that creates an overabundance
of CD8+ T lymphocytes. It appears to have a partial penetrance, autosomal dominant
inheritance pattern. Further analysis has shown that these cells may be impairing fracture
healing. Your colleagues want to meet with you to discuss enrolling patients in a clinical
study to identify genetic “hot spots” that may represent loci with allelic variants that
distinguish good fracture healers from poor healers. The study design most likely to
identify the hot spots is

1. whole-exome sequencing.
2. genome-wide association.
3. prospective cohort database generation.
4. DNA microarray.

Discussion: 2

A case-matched control study (for more rare events) and prospective databases (for more
common events) may help identify factors associated with good healing and poor healing.
Used alone, they do not help identify genetic information related to the phenotype,
however. Although whole-exome sequencing may identify variations or mutations in the
coding regions of DNA, it will not find them outside of those regions. Therefore, when
the nature of the mutation or gene variation is unknown, a genome-wide association is
most likely to find hot spots associated with the phenotype. DNA microarray is useful f or
quantifying specific genes of interest.

Question 80 of 100
A clinical study enrolls patients into two groups to analyze the effects of smoking on
patient-reported outcomes following elective spinal diskectomy. Patients are to be
blinded and enrolled consecutively. The study data are analyzed after enrolling half the
number of patients estimated to be needed by the power analysis. No statistically
significant differences are found. Accepting the results at this point would be an example
of

1. selection bias.
2. type 1 error.
3. type 2 error.
4. intent to treat.

Discussion: 3

Type 2 error occurs when a clinical trial is underpowered to detect the treatment outcome
differences between two groups. Type 1 error occurs when a false positive effect is
detected. Selection bias refers to failure to properly randomize to achieve a representative
sampling of the population. Intent to treat is a method of analysis based on initial
allocation.
Question 81 of 100
CLINICAL SITUATION

A study evaluating hip arthroscopy for labral tears enrolls more than 200 participants,
who are randomized into two groups: surgical and nonsurgical care which included
physical therapy and NSAIDs. The primary endpoint was a composite consisting of: 1)
the patient reporting change between preintervention pain as reported on the 10-point
visual analog scale and the postintervention score at 2 years, and 2) absence of major
adverse events, including death, return to the operating room, and major neurologic
injury.

What is currently considered the standard method of managing author conflicts of


interest?

1. Rejection of all industry investments, consultation fees, and funding


2. Acceptance of funds used only for research purposes
3. Disclosure of relevant potential conflicts of interest
4. Review of results by the sponsors before publication

Discussion: 3
The goal of this series of questions is to test the knowledge of study design. Poolman and associates
have described four parties who can be subject to blinding, and it is important to clearly state in the
study methodology who is blinded because of the various types ofbias. In this case, because of the
nature of the study design, only the data analysts are able to be blinded practically .

Allocation concealment is an aspect of study design that can be applied to any clinical trial. It is
important to prevent the treating physician from being able to guess the group to which the next
subject will be randomized. Otherwise, an inherent risk of selection bias is present. All the methods
are predictable except randomization with opaque envelopes.

Physician-driven outcomes are a concern in research. In such instances, the physician conducting the
study also determines when the variables he or she is measuring are met. This practice leads to
obvious clinical bias. One way of helping to reduce such bias is to set strict, a priori definitions of
treatment success and failure, leaving less room for subjectivity.

Currently, the handling of conflicts of interest is a challenging problem. Alt hough some
groups support stricter options, the existing standard is for the physician to openly disclose any real
or potential conflicts of interest.
Question 82-87 of 100
For the following external fixation principle, match the most likely contributing factor
listed

1. Pin diameter
2. Hydroxyapatite coating
3. Increased bone-to-rod distance
4. Thread-shank junction
5. Increased pin spread
6. Excessive motion of muscle and/or skin around the pin
7. Pin-clamp junction

When applying an external fixator, the surgeon always must consider the risk -benefit
ratio between stability and potential complications.

The weakest point of a fixation pin Answer : 4

Pinsite Infection Answer: 6


Provides greater extraction torque of fixation pins Answer: 2

Increases construct stability Answer: 5

Decreases construct stability Answer: 3

Contributes to stress riser formation Answer: 1

Discussion:

The application of external fixation always involves a risk-benefit evaluation that


includes the key considerations of stability, soft-tissue protection, and the potential
forfuture surgery. An external fixation device may be constructed in many ways, but
certain principles are to be considered. The ideal construct for stability generally
consists of one pin as close to the fracture site as possible and another pin as far away
as possible in the same bone. Stability can be enhanced by increasing the pin diameter,
the number of pins used, the pin spread, the number of planes of fixation, the diameter
of the rods, and the number of rods and by decreasing thebone-to-rod distance. A
fixation pin’s weakest point is the thread-shank junction. Burying the shank into the
proximal cortex can double the pin’s stiffness. Hydroxyapatite-coated pins have been
shown to improve the pin-bone interface and require greater extraction torque. It is
thought that excessive motion of muscle and skin around the pin results in local
inflammation, leading to pin tract infections. This riskcan be reduced by using a gentle
compressive dressing around the pin that serves as a bolster between the skin and the
clamp.

Question 88-91 of 100

Select the appropriate material property for the definition listed.


1. Brittle
2. Hysteresis
3. Viscoelastic
4. Yield point
5. Elastic deformation
6. Plastic deformation

Changes in a material’s shape because of load; it is reversible when the load is removed
Answere: 1

Changes in a material’s shape that is irreversible after the load is removed


Answer: 6

When a loading curve does not match the relaxation curve


Answer: 2

The amount of strain required to transition from elastic to plastic deformation


Answer: 4
Discussion:

Materials exhibit a different stress-strain curve during loading based on their inherent
properties. As a load is applied, the curve can be linear (brittle material), can undergo
plastic deformation (ductile material), or can vary based on the rate of the load applied
(viscoelastic material). As the material relaxes, it may or may not return to its original
shape. Hysteresis describes a material whose relaxation curve does not match the
loading curve. A material that is able to return to its original shape is said to have elastic
deformation, whereas plastic deformation is permanent. The transition from elastic to
plastic deformation is the yield point at which the material will no longer return to its
original shape.

Question 92- 96 of 100


Match each soft-tissue fixation method with its most common mode of failure.

1. Interference screw
2. Staple
3. Suture anchor
4. Suture post
5. Transfixion pins
6. Cortical button

Tendon slippage Answer: 1

Device pullout Answer: 2

Device breakage Answer: 5

Pulling through bone Answer: 6

Tendon Laceration Answer: 1


Discussion:

The surgical fixation of soft tissue to bone is a common orthopaedic procedure.


Numerous factors influence the strength of the soft–tissue-to-bone fixation construct,
including tissue quality, implant strength, contact area, tensioning, and the biology of
the location. Each fixation technique differs with respect to stability and failure
mechanism. Interference screws can achieve excellent fixation but have the potential
for graft pullout or laceration. Staples have less stiffness than interference screws and
can irritate the overlying tissues. Transfixion pins fail secondary to pin breakage or
partial tearing of the graft. Primary failure of cortical buttons includes pulling through
cortical bone or migration.

Question 97- 100 of 100


Match the stain or immunoprobe with the method of investigation for identifying the
process.

1. Hematoxylin and eosin


2. Parathyroid–hormone-related protein immunostain
3. Safranin-0
4. Cathepsin-K immunostain

Examination of cartilage in early callus formation Answer: 3

Determining osteoclast activity in bone remodeling Answer: 4

Growth plate signaling evaluation Answer: 2

Osteomyelitis evaluation Answer: 1


The hematoxylin and eosin stain is used very broadly in standard histology, such as to
evaluate for the presence of polymorphonuclear cells in infection or osteomyelitis.
Hematoxylin makes nuclei blue and therefore easier to identify. Cartilage and
proteoglycan are evaluated well with safranin-O and alcian blue histology stains. The
differentiation of osteoclasts, as when attempting to evaluate how briskly remodeling of
bone is occurring, could employ tartrate-resistant acidic phosphatase histology staining
or cathepsin-K immunostaining. Cell signaling, between parathyroid–hormone-related
protein (PTHrP) and indian hedgehog protein in the growth plate has been evaluated by
Immunostain for PTHrP

You might also like